You are on page 1of 340

English for

Law

Thomas
Gaultier
• Presentations

As we will be spending quite some time


together…
… it’s important to know one another.

– Name
– Favourite Hobby
– …

2
• Presentations

Who am
I?

3
• Learning objectives of this course

– Upon completion of this course, students will have/be:


• Acquired substantial skills in using technical English as
lawyers
• Capable of thinking about law in English
• Capable of understanding and discussing a US court
decision
• Capable of applying legal reasoning in English to a set of
facts

We will be doing so by studying the area of TORT law


4
• Torts (Judge Judy / the
NEWS!)

5
• Program of the
Semester

– Torts
• Study of the main concepts of tort law
• Analysis and discussion of court
decisions

6
• Assessment (I)
– Ongoing in-class assessment : 50% of the final grade
• Students will be expected to each give an oral presentation
on a court decision in class (worth 50% of their ongoing
assessment grade).
• Students will have to complete a take-home assignment on
intentional torts (worth 50% of their ongoing assessment
grade).

– Students are eligible for this type of assessment only if they


have completed both elements of the ongoing-assessment.

7
• Assessment (II)
– Final exam: 50% of final grade
• The final exam will consist of
– 10 multiple choice questions
– 1 IRAC (issue spotting) case study
» The objective is to apply legal reasoning to a set of
facts, using the appropriate terms, concepts,
vocabulary and rules of law
– The final written exam will be open book!
– Should the final exam grade be higher than the ongoing
assessment grade, the final grade of the course will be that
of the final exam. 8
• A few rules
– Mutual respect!
• As you will be interacting and participating a lot (I hope),
respect your fellow students.
• There are no stupid questions in this class – only opportunities
to learn more
• When others are speaking, please don’t interrupt. If you all
speak at the same time, it will be a mess.
– Breaks: 15 min halfway through the session.
• If we all come back from break on time, the session will
finish on time!

9
• Class Preparation
– Court decisions
• Students are expected to come to class prepared to discuss
the court decisions that will be given as assignments.
• Students scheduled to give an oral presentation on these
court decisions should be prepared to do so.

10
Questions
?

11
Presentation Schedule
You will need to sign up online
(more information will be made available
shortly)

12
TORT
S
Main question: Is this defendant going to be liable to
this plaintiff for the tort at issue?

Subquestion 1: Can the plaintiff prove every elements of


the tort?
Subquestion 2: Are there any affirmative defenses to this tort?

13
Main legal sources for interpreting torts:

1) RESTATEMENT OF TORTS
2) CASE LAW

14
PART A. INTENTIONAL TORTS AND PRIVILEGES

Chapter 1

INTENTIONAL INTERFERENCES WITH PERSONS OR PROPERTY

15
§ 1.01 Intent

[A] Overview and Definition

Intentional torts share the requirement that the defendant


intentionally commit the elements that define the tort. Most
contemporary courts define intent to mean either that the
defendant desires or is substantially certain the elements of the
tort will occur.

16
Example: Frustrated Ballplayer throws his bat into the stands. If
the bat hits Spectator, can we say that Ballplayer “intended” the
contact?

17
[B] Transferred Intent

Under the transferred intent doctrine, accepted by many courts,


intent can be transferred between five torts (battery, assault,
false imprisonment, trespass to chattel, and trespass to land)
and different victims within these five torts. For example, if A
intends to assault B, but accidentally commits battery against B
or another party C, A is liable for the battery.

Intent can “transfer” from intended to actual victim


Intent can “transfer” from intended to actual tort

18
Example: Carl leaps over Neighbor’s fence, intending to land on
the lawn. Instead, Carl lands on Neighbor’s Fiancé. Can Fiancé
prove intent in a battery action against Carl?

19
[C] Insanity and Infancy

Unlike in criminal law, neither insanity nor infancy is a defense


for an intentional tort. However, intent is subjective and requires
that the defendant actually desires or be substantially certain
the elements of the tort will occur. Consequently, if the
defendant is extremely mentally impaired or very young, she
may not actually possess the requisite intent.

20
§ 1.02 Battery

[A] Overview and Definition

Battery occurs when the defendant’s acts intentionally cause


harmful or offensive contact with the victim’s person. (Accidental
contact, by contrast, must be analyzed under negligence or strict
liability).

NOT limited to body—includes anything that P might be holding


or that is connected to P’s body!
21
Example: A fellow student taps you on the shoulder at our break,
asking for the location of the restrooms. Is the contact
“unpermitted?”

22
[B] Intent Requirement

While battery requires intent, the prevailing tort definition does


not require an intent to harm. It is only necessary that the
defendant intend to cause either harmful or offensive contact.
The transferred intent doctrine is applicable to battery.

23
[C] Harmful or Offensive Contact

Battery encompasses either harmful or offensive contact.


Even trivial offensive contact can constitute a battery.

Can you think of any examples?

24
[D] Causation

The defendant's voluntary action must be the direct or indirect


legal cause of the harmful or offensive contact. However,
defendant need not herself actually contact the victim.

25
Example: Defendant knocks a plate out of Plaintiff’s hand. Is
there contact with Plaintiff’s “person?”

26
Example: Defendant shakes Plaintiff’s car while Plaintiff is inside.
Is there contact with Plaintiff’s “person?”

27
§ 1.03 Assault

A. Overview
The ancient tort of assault represents the still controversial
recognition that pure psychological injury should be
compensable.

B. Definition
Assault occurs when the defendant's acts intentionally cause the
victim's reasonable apprehension of immediate harmful or
offensive contact.
28
1 Intent Requirement

Assault is an intentional tort. The defendant must desire or be


substantially certain that her action will cause the apprehension
of immediate harmful or offensive contact. The transferred
intent doctrine is applicable to assault.

2 Apprehension

The victim must reasonably perceive that harmful or offensive


contact is about to happen to him.
29
Example: Bob reaches toward his back pocket for his wallet. You
fear that Bob is pulling out a gun. Reasonable apprehension?

30
Example: Bob points an unloaded gun at you. Does this create
reasonable apprehension despite Bob’s lack of actual ability to
shoot?

31
[3] Imminent Harmful or Offensive Contact

For assault to be actionable the victim's apprehension must be


of imminent harmful or offensive contact.

32
Example: Homer says, “If you don’t promise me a job, I’m going
to punch you in the mouth.” Immediacy?

33
[4] Fear versus Apprehension

Several court decisions distinguish between "fear” and


“apprehension.” The requisite apprehension of imminent
contact need not produce fear in the victim.

34
Example: Weakling tries to punch Professional Wrestler. Does
Wrestler “reasonably apprehend” an “immediate contact” even
if he is not scared or intimidated?

35
[5] Words

Words alone are not enough. Assault requires some


accompanying physical conduct.

Words alone are NOT enough, BUT words may


negate the threat.

36
Example: Homer shakes his fist under your chin while saying, “If
you weren’t my best friend, I’d punch you in the mouth.” Is there
reasonable apprehension of immediate contact?

37
§ 1.04 False Imprisonment

[A] Overview and Definition

In false imprisonment, the defendant unlawfully acts to


intentionally cause confinement or restraint of the victim within
a bounded area. There must be no reasonable or apparent
means of escape.
Accidental confinement is not included and must be addressed
under negligence or strict liability. The transferred intent
doctrine is applicable to false imprisonment.

38
Example: Plaintiff can leave through a rat-infested sewage pipe.
Is this a “reasonable means of escape?”

Example: Plaintiff can leave through a hidden passageway in a


wall. Is there a means of escape of which Plaintiff is “aware?”

39
[B] Bounded Area

The victim must be confined within an area bounded in all


directions. The bounded area can be, however, a large area,
even an entire city.

40
Example: A barricade forces you to walk around the block.
Bounded area?

41
[C] Means of Confinement or Restraint

For false imprisonment to exist, the victim must be confined or


restrained. The confinement may be accomplished by:
(1) physical barriers;
(2) force or threat of immediate force against the victim, the
victim's family or others in her immediate presence, or the
victim's property;
(3) omission where the defendant has a legal duty to act; or
(4) improper assertion of legal authority.

42
Example: Religious Colony Leader takes Plaintiff across the ocean
but refuses to give her the means to come ashore. Is Leader’s
inaction a “sufficient act of restraint?”

43
[C] Means of Confinement or Restraint (continued)

The improper assertion of legal authority can unlawfully restrain


a victim. This form of false imprisonment constitutes false arrest.
The victim must submit to the arrest for it to constitute
imprisonment. The arrest is improper if the actor imposing
confinement is not privileged under the circumstances.

44
[D] Consciousness of Confinement

False imprisonment requires that the victim be conscious of the


confinement at the time of imprisonment. The Restatement
modifies this requirement and would find liability for false
imprisonment, even when the victim is not aware of the
confinement, if the victim is harmed by the confinement.

45
§ 1.05 Trespass to Chattel and Conversion

[A] Overview

Trespass to chattel and conversion are two separate intentional


torts that protect personal property from wrongful interference.
The two torts, which overlap in part, are derived from different
historical origins. In many, but not all instances, both torts may
be applicable.

46
[B] Definition of Trespass to Chattel

Trespass to chattel is the intentional interference with the right


of possession of personal property. The defendant's acts must
intentionally damage the chattel, deprive the possessor of its use
for a substantial period of time, or totally dispossess the chattel
from the victim.

Example: D takes P’s car for a five-minute “joy ride,” and returns
it unharmed. D has committed a trespass to chattel.

47
[B] Definition of Trespass to Chattel

Trespass to chattel does not require that the defendant act in


bad faith or intend to interfere with the rights of others. It is
sufficient that the actor intends to damage or possess a chattel
which in fact is properly possessed by another.

Trespass to chattels is about possession (usually requires


damage).

Conversion is about usage (does not require damage).


48
[C] Definition of Conversion

The Restatement defines conversion as “an intentional exercise


of dominion and control over a chattel which so seriously
interferes with the right of another to control it that the actor
may justly be required to pay the other the full value of the
chattel.”

49
Example: D steals P’s car, then seriously (though not irreparably)
damages it in a collision. Is D liable for conversion?

50
Conversion is an intentional tort, but all that is required is that D
have intended to take possession of the property. Mistake as to
ownership will not be a defense.

Example: D buys an old painting from an art dealer, and


reasonably believes that the art dealer has good title. In fact, the
painting was stolen from P years before. D keeps the painting in
his house for 10 years. D is liable for conversion, notwithstanding
his honest mistake about title.

51
Transfer to third party: D can also commit conversion by
transferring a chattel to one who is not entitled to it.

Example: D, a messenger service, delivers a package to the


wrong person, X. X absconds with the goods. D has committed
conversion, even though D did not end up with possession of the
goods.

52
Withholding good: D may commit conversion by refusing to
return good to their owner.

Example: D, a parking garage, refuses to give P back her car for a


day.

The essence of the conversion claim is that the defendant has


exercised dominion over the goods. There is generally no liability
for conversion until the plaintiff has demanded return of the
chattel and has been refused.

53
§ 1.06 Trespass to Land

[A] Overview

Trespass protects a plaintiff's interest in the surface land itself,


the earth or other material beneath the surface, and “the air
space above it.” Depending upon the seriousness of the
contamination of plaintiff's land or environment, defendant may
be liable in trespass where the pollution or contamination
interferes with plaintiff's possessory rights in the land, the land
beneath it, or the ambient air.

54
Example: American Airlines jet flies over Blackacre at 35,000
feet. Trespass to Blackacre?

55
Example #2: Kid throws ball across Neighbor’s yard, it lands of
the other side, but further than the Neighbor’s property
boundary. Trespass to Neighbor’s yard?

56
Restatement provides:

One is subject to liability to another for trespass, irrespective of


whether he thereby causes harm to any legally protected
interest of the other, if he intentionally (a) enters land in the
possession of the other, or causes a thing or a third person to do
so, or (b) remains on the land, or (c) fails to remove from the
land a thing which he is under a duty to remove.

57
Example: D, a pilot, loses control of the aircraft, and the aircraft
lands on P’s property.
Is this Trespass to land?

58
[B] The Requirement of Intent

Trespass is an intentional tort, and plaintiff must prove that an


alleged trespasser had the requisite mental state to commit the
tort. Restatement states that the word “intent” is used in tort
law “. . . to denote that the actor desires to cause the
consequences of his act, or that he believes that the
consequences are substantially certain to result from it.”

59
Example: Bob wanders onto private property while hiking
through state park. Act of physical invasion?

60
[C] The Requisite Physical Invasion and Harm

In most jurisdictions, invasions of plaintiff's property that


amount to trespass may also, if they interfere with plaintiff's use
and enjoyment of the property, be actionable in nuisance. In
such circumstances, “plaintiff may have his choice” of a claim in
trespass or in nuisance, “or may proceed upon both.”

61
At common law, an actionable trespass was complete upon the
tangible invasion of another's property. Nuisance, in contrast,
requires a showing that defendant's conduct, invasory or
otherwise, constitutes a “substantial and unreasonable”
interference with plaintiff's use and enjoyment.

That the plaintiff may be entitled to an injunction or to nominal


damages for a technical or minimal trespass does not resolve the
question of what level of harm plaintiff must show to receive
compensatory damages. It will be up to the trier of fact to
determine how to quantify the damages.

62
§ 1.07 Nuisance

[A] Overview
Nuisance arises from an allegation of injury to person or
property. As in other areas of tort, the injury need not be
physical, and can include injury to rights or property enjoyment.
The law of nuisance recognizes two distinct categories of claims:
private nuisance and public nuisance. Defendant's conduct may
create an actionable public nuisance when it interferes with the
public health, safety or welfare. It may constitute a private
nuisance when it interferes with another's current possessory or
beneficial interest in the use or quiet enjoyment of land.
63
[B] Nuisance and Trespass Distinguished

A claim in trespass ordinarily seeks damages for a physical


intrusion onto property.
Where the intrusion is permanent, or if it is serious or persistent,
the suit sounds in trespass.
In contrast, when the defendant's conduct creates conditions of
noise, lights, odor or vibration that interfere with the plaintiff's
quiet enjoyment of the property, but do not interrupt the
plaintiff's possessory interests, the claim is more properly
brought in private nuisance.

64
C. Private Nuisance

1 Elements

a. Unreasonable Interference

A private nuisance is an “unreasonable interference” with the


use or enjoyment of the owner or possessor's use or enjoyment
of a property interest. It might take a wide range of forms, from
the pollution of a residence's well water; or the creation of a
sulphurous smell within a downtown financial district.

65
[b] Intentional or Unintentional Conduct

For a defendant's act to be “intentional”, he need only have


intended the act. When the act is “unintentional”, or accidental,
liability only follows a showing that the defendant's conduct was
negligent, reckless, or constituted an abnormally dangerous
activity.

In nuisance actions, the intentional character of a defendant's


conduct may be proved circumstantially.

66
[2] Nature of the Interest Interfered with

Some decisions distinguish nuisance per se from nuisance per


accidens. A nuisance per se would be any act that constitutes a
nuisance under any circumstances, such as the permanent
chronic contamination of plaintiff's property, or the maintenance
of a house of ill repute.

Nuisance per accidens requires the fact finder's evaluation of


whether, “under all the surrounding circumstances . . . [the acts]
substantially interfer[e] with [plaintiff's] comfortable
enjoyment[.]”
67
D. Public Nuisance

1 Generally

Public nuisance is defined widely as “an unreasonable


interference with a right common to the general public.”

68
[2] Proper Complainants

A public nuisance suit may be brought by a public official or a


public agency, or it may be brought by a private individual or
business that has “suffered harm of a kind different from that
suffered by other members of the public.”

For the most part, the public nuisance remedy is enforced by a


government body, such as a town, on behalf of the public. For
example, a municipality might lodge a claim in public nuisance
against a manufacturing facility discharging chemical effluent
that is contaminating a nearby lake, killing aquatic life and
precluding recreational sports and swimming. 69
[3] Special Injury Rule

While suits in public nuisance are usually brought by public


bodies, such as a state or a political subdivision, under certain
circumstances a private individual may sue in public
nuisance.

An individual may sue another in public nuisance where he


proves that there is a substantial interference with a right
common to the public, and additionally, that he has suffered
special harm that differs in type or quality from that burdening
the public.
70
[4] Environmental Harm

Public nuisance is a potentially potent claim for governmental


bodies seeking to interdict generators and disposers of
hazardous waste.

71
§ 1.08 Intentional Infliction of Mental Distress

[A] Definition

Intentional infliction of mental distress exists when the


defendant, by extreme and outrageous conduct, intentionally or
recklessly causes the victim severe mental distress. Most states
no longer require that the victim suffer physical manifestations
of the mental distress.

72
§ 1.08 Intentional Infliction of Mental Distress

[A] Definition (continued)

The Restatement defines extreme and outrageous conduct as


behavior which is “beyond all possible bounds of decency and to
be regarded as atrocious, and utterly intolerable in a civilized
community.” The vulnerability of the victim and the relationship
of the defendant to the victim can be critical.

73
Example: D, as a practical joke, tells P that her husband has been
badly injured in an accident, and is lying in the hospital with
broken legs. Does this qualify as outrageous?

74
[A] Definition (continued)

Conduct that is not normally outrageous may become so if


(factors to consider):
• Conduct occurs in public (rather than private) (i.e.,
public humiliation)
• It is continuous or repetitive in nature (i.e., ongoing insults…
hours, days, weeks… the conduct becomes outrageous)
• It is directed towards a member of a "fragile class", the conduct
can become outrageous
– Very young child
– Elderly person
– Pregnant 75
[B] Intent or Recklessness to Cause Severe Mental Distress

For recovery under intentional infliction of emotional distress,


the plaintiff must prove that the defendant intended to cause
severe emotional distress or acted with reckless disregard as to
whether the victim would suffer severe distress. Although
characterized as an “intentional” tort, recklessness, in addition
to intent, generally suffices for liability.

76
Example: D commits suicide by slitting his throat in P’s kitchen. D,
or his estate, is liable for intentional infliction of emotional
distress because although D did not desire to cause distress to P,
or even know that the distress was substantially certain, he
recklessly disregarded the high risk that distress would occur.

77
[C] Third Party Recovery

Generally, there is no transferred intent for IIED.

Example: Defendant beats Man while random Pedestrian


watches from across the street. Does Defendant’s intent to
batter Man transfer to Pedestrian’s intentional infliction of
emotional distress action?

78
[C] Third Party Recovery

However, courts have usually awarded a third-party victim


recovery only if, in addition to proving the elements of the tort,
she is:
(1) a close relative of the primary victim;
(2)present at the scene of the outrageous conduct against the
primary victim; and
(3) the defendant knows the close relative is present.

79
Example: Defendant beats Man while Best Friend watches from
across the street. Does Defendant’s intent to batter Man transfer
to Best Friend’s intentional infliction of emotional distress
action?

80
[D] Exception for Common Carriers and Innkeepers

Common carriers and innkeepers are liable for intentional gross


insults which cause patrons to suffer mental distress. There is no
requirement the defendant behave in an extreme or outrageous
manner or that the victim suffer extreme distress.

81
Example: Hotel Clerk calls Bob, who is a hotel client, a “smelly,
pig-like creature.” Can Bob maintain an action for intentional
infliction of emotional distress?

82
Example: Hotel Clerk insults Pedestrian. Can Pedestrian maintain
an action for intentional infliction of emotional distress?

83
Chapter 2

PRIVILEGES TO INTENTIONAL TORTS

84
§ 2.01 Consent

[A] Overview

Consent is a defense to intentional tort liability. If the asserted


victim gives permission, what would otherwise be tortious is
instead privileged.

85
[B] Express and Implied Manifestations of Consent

An individual can convey consent expressly in words or through


pictorial gestures. Alternatively, an individual can imply consent.
Consent is implied when, under the circumstances, the conduct
of the individual reasonably conveys consent.

86
Example: P says to D, “Go ahead, hit me in the stomach – I’ll
show you how strong I am.” If D does so, P’s consent prevents P
from suing for battery.

Example: D offers to vaccinate all passengers on their ship. P


holds up her arm and receives the vaccination. Since it
reasonable appeared to D that P consented, there will be
consent regardless of P’s actual state of mind.

87
[C] Consent by Law

Consent can also be implied by law. Generally courts recognize


by law consent to emergency medical treatment by health
professionals when a victim is unconscious and unable to
provide consent.

Example: P is brought unconscious to the emergency room of D,


a hospital. D can perform emergency surgery without P’s actual
consent – consent will be implied as a matter of law. Therefore, P
cannot sue for battery.

88
D. Invalidating Manifestations of Consent

1 Incapacity

Both express and implied manifestations can be held invalid.


An individual can be held to lack capacity to consent. A
child, depending on her age, may consent only to less
significant matters.

An individual without sufficient mental capacity due to insanity


or retardation may not legally consent. Incapacity can also be the
result of drug ingestion (including alcohol).
89
[2] Action Beyond Scope of Consent

Consent is also invalidated if the action goes beyond the consent


manifested. What constitutes the dimensions of the consent can
often be a different issue of fact.

Example: P visits D, a doctor, and consents to an operation on


her right ear. While P is under anesthetic, D decides that P’s left
ear needs an operation as well, and does it. P’s consent does not
block an action for battery for the left-ear operation, since the
operation went beyond the scope of P’s consent.

90
Example #1: Homer is tagged during a touch football game. He
sues his opponent for battery, arguing that the tag was an
“unpermitted contact.” Can Homer’s opponent successfully use
the defense of implied consent?

91
Example #2: Homer was just off the boat from Portugal and
doesn’t know the rules of touch football. Does this change the
result?

92
Example #3: The opponent kicks Homer’s groin for good
measure. Has the opponent exceeded the boundaries of his
privilege?

93
[3] Fraud

Consent is invalid if it is induced by fraud that misrepresents an


essential aspect of the interaction.

Example: Claude, the owner of a spa, offers to give Julia a mud


bath. She says: “Sure, go right ahead!”. However, the mud was
filled with dog poop. Did Julia consent to the offensive touching?

94
[4] Duress

Consent procured under physical threat is invalid. However, as a


general rule, economic pressure, while coercive, does not negate
consent.

[5] Illegality

The traditional majority rule holds that a person cannot consent


to a criminal act; the consent is always invalid.

95
§ 2.02 Self-Defense

[A] Overview and Definition

Self-defense constitutes a defense which can justify and


therefore negate intentional tort liability. In essence, reasonable
force can be used where one reasonably believes that such force
is necessary to protect oneself from immediate harm.

96
B. The Threat Must be Immediate

Self-defense must be in response to an immediate threat of


harm. Or else, it is simply revenge.

C. The Victim's Response Must be Reasonable

Self-defense is only justified if the individual reasonably believes


that force is necessary to avoid an unlawful attack. The belief
need not be correct, however.

97
Example: Bob sprays mace at a man who reaches into his pocket
on a street corner. Did Bob have a “reasonable belief” that he
was in danger?

98
Force intended to inflict death or serious bodily injury is only
justified if the individual reasonably believes she would suffer
serious bodily injury or death from the attack.

Example: Bob throws wild punches at you. Can you shoot Bob in
self- defense?

99
[D] The Obligation to Retreat From Deadly Force

There is general agreement that there is no obligation to retreat


from force not threatening death or serious bodily injury.

100
There is disagreement among jurisdictions whether retreat is
required where self-defense would require the use of force
intended to inflict serious bodily injury or death. The traditional
and still majority common law position does not require
retreat, assuming the threatened individual has the legal right
to be present or to proceed.

The minority position, endorsed by the Restatement, requires


retreat where serious bodily injury or death would otherwise be
required in self-defense. The minority position would not,
however, require retreat from the victim's dwelling.

101
§ 2.03 Defense of Others

[A] Overview

A person can use reasonable force to protect a third person from


immediate unlawful physical harm.

The modern trend and now majority view holds there is a


privilege to use reasonable force to protect a third party
whenever the actor reasonably believes a third party is entitled
to exercise self-defense.

102
Example: Chivalrous Man strikes Plainclothes Policeman, who is
arresting a young woman. Can Chivalrous Man assert the
defense of others?

103
§ 2.04 Defense and Recovery of Property

[A] Overview

An individual is privileged to use reasonable force to prevent a


tort against her real or personal property. However, unlike self-
defense, a reasonable mistake will not excuse force that is
directed against an innocent party.

104
[B] Reasonable Force

Only reasonable force can be exercised in protection of property.


Force intended to inflict death or serious bodily injury is never
reasonable to protect merely property.

Even slight force is unreasonable in defense of property if it is


excessive. Consequently, if a verbal request would suffice, no
force is justified.

105
Example: Sandy shoots Burglar in her bedroom. When Burglar
sues Sandy for battery, what defense may Sandy properly raise?
Defense to property?

Self-defense?

106
[C] Defense of Habitation

The modern view is that the use of deadly force or force likely to
cause serious bodily harm is not justified unless the intruder
threatens the occupants' safety, by committing or intending to
commit a dangerous felony on the property. Additionally, the
homeowner may not eject a non-threatening trespasser or
invited guest when doing so would subject that person to serious
physical harm.

107
Example: D sees P trespassing in D’s backyard. D asks P to leave,
but P refuses. Even if there is no way to make P leave except by
shooting him, D may not do so, since P’s conduct does not
threaten D with death or serious bodily harm.

108
[D] Mechanical Devices

Intentional mechanical infliction of deadly force, such as by the


use of spring guns, is not privileged unless, in fact, such force
was justified to defend oneself or another from deadly force.

Barbed wire fences and similar deterrents to enter land


unlawfully are not generally perceived as intended to inflict
death or serious bodily injury but are often designed only to
deter entry.

109
[E] Recovery of Personal Property

An individual may use reasonable force to recover property


when in “hot pursuit” of the wrongdoer.

Example: A learns that B has stolen a stereo and is in possession


of it. A may use reasonable force to reclaim the stereo if he acts
immediately, but not if he waits, say, a week between learning
that D has the property and attempting to regain it.

110
The individual acts at her peril, however. If force is directed at an
innocent party, privileged to possess the chattel, or against one
acting out of a bona fide claim of right, whether or not such right
is ultimately vindicated by the courts, the actor is liable even if
the mistake was reasonable.

Example: O rents a TV to A. A refuses to return the TV to O on


time. O probably may not use reasonable force to enter A’s
home and repossess the set, because A’s original possession was
not wrongful.

111
Many states have adopted a merchant's privilege, which allows
stores to use reasonable force to detain a person for reasonable
periods to investigate possible theft. The merchant's privilege
generally allows reasonable mistake, so an innocent customer
cannot recover against the store, provided the store acted
reasonably.

112
§ 2.05 Necessity

[A] Overview and Definition

Necessity is a defense which allows the defendant to interfere


with the property interests of an innocent party in order to avoid
a greater injury. The defendant is justified in her behavior
because the action minimizes the overall loss. The defense is
divided into two categories: public and private necessity.

113
Example: Jerry has his fist raised, ready to hit Homer. Homer hits
Jerry as a preemptive strike. Jerry sues Homer for battery. Can
Homer successfully use the defense of necessity?

114
[B] Public Necessity

Public necessity exists when the defendant appropriates or


injures a private property interest to protect the community.
Public necessity is a complete defense.

Example: Mayor orders destruction of row houses to stop fire


from spreading through the city. Is Mayor’s action privileged as a
public necessity?

115
[C] Private Necessity

Private necessity exists when the individual appropriates or


injures a private property interest to protect a private interest
valued greater than the appropriated or injured property. Private
necessity is an incomplete defense: the defendant is privileged
to interfere with another's property, but is liable for the damage.

116
Example: Charles lands his private plane on Farmer Jones’
soybean field when his plane experiences engine trouble. When
Farmer Jones sues for trespass to land...

Will Charles be able to assert the defense of private necessity?

Will Charles be responsible for the damages to Farmer Jones’


crops?

117
NEGLIGENCE

Chapter 3
THE NEGLIGENCE CONCEPT AND THE REASONABLE PERSON
STANDARD OF CARE

118
General Philosophy: In our life, we are always creating risk.

It’s the Star Trek Theory that we are always emanating “risk
rays” from our bodies. They may vary in size and intensity
depending on the activity the individual is engaged. There is
always risk around us, even though it is sometimes very unlikely.
On some occasions the risk rays become more powerful, when
we are driving for example, when we are using dangerous
machines or materials…
The principle of negligence says: Before you act, take some
precaution to dampen/reduce those “risk rays”!

119
§ 3.01 Overview

To recover for negligence, the plaintiff must establish each of the


following elements by a preponderance of the evidence (that is,
by more than 50%) to establish a prima facie case: duty,
standard of care, breach of duty, cause-in-fact, proximate cause
(scope of liability) and damages.

120
Example #1: Joe drives down the street at 90 m.p.h. when he hits
Pedestrian. Is Pedestrian a foreseeable victim?

121
Example #2: Unforeseeable Plaintiff: Ms. Jones is hit on the head
by a branch that was blown 10 meters away due to an
inadvertent fireworks explosion that was the result of a man
being tripped over a sidewalk by a prankster while carrying a bag
containing fireworks. Is Ms. Jones a foreseeable plaintiff?

122
§ 3.02 Standard of Care

The standard of care is the level of conduct demanded of a


person so as to avoid liability for negligence. Failure to meet this
standard is characterized as breach of duty.

Must ask two basic questions:


- To whom did D owe a duty (was P foreseeable)?
- What is the applicable standard of care?

123
§ 3.03 The Reasonable Person

The most common standard of care in negligence law commands


the defendant to act as would a reasonably prudent person in
the same or similar circumstances.

If the defendant does so, she is protected from negligence


liability. Failure to do so constitutes unreasonable conduct and,
hence, breach of duty.

124
§ 3.03 The Reasonable Person (continued)

The reasonable person standard is an objective standard that


compares the defendant's conduct to the external standard of a
reasonable person. Thus, the law imposes on each person of
society an obligation to conform to the objective reasonableness
standard.

Debate centers around the extent to which there is flexibility in


this objective standard.

125
[A] The Characteristics of the Reasonable Person

The reasonable person possesses those attributes that a jury


decides represent the community norms.
The reasonable person is no real person. While the reasonable
person's qualities are those the jury determines are the
expected attributes of those in the community, the reasonable
person cannot be expected to be infallible.
Further, the objective standard assumes that the reasonable
person possesses the general experience of the community.
It is the knowledge and understanding generally held by
members of the community that is relevant.
126
Example: Is Motorist expected to use “extreme care” while
driving in a blizzard?

127
[B] Flexibility in the Reasonable Person Standard

Using the standard of the reasonable person under the same or


similar circumstances, flexibility can be added through the
“circumstances” part of the analysis.
Ultimately, in most jurisdictions, a jury will be permitted to
consider the physical conditions of the defendant and that the
defendant was acting under emergency conditions.
Most other characteristics, such as mental conditions or
inexperience, are not taken into account.

128
[1] Emergency

Nearly all states permit the jury to consider in its determination


of the defendant's reasonableness evidence that the defendant
was acting under emergency conditions not of the defendant's
making.
The fact that the defendant was acting in an emergency does not
necessarily exculpate the defendant from liability.

129
[2] Physical Conditions

As a general principle, because they are easily measured and


perceived as tangible, the defendant's own physical qualities
may be taken into account by the jury in the breach of care
determination.
Just because the party's physical condition is taken into account
does not mean that she will be exonerated, however, as
sometimes the physical condition of the party requires the use
of greater care.

130
[3] Mental Conditions

Most courts treat mental conditions as wholly irrelevant for


purposes of negligence liability. The insane are held to a
standard of sanity and people with cognitive disabilities are held
to a level of normal intelligence.

131
[4] The Effect of Superior Abilities, Skill or Knowledge

The standard of care does not change for those with superior
skills although the defendant's special skills (doctor, accountant,
lawyer, etc..) may affect the jury's breach determination. The
reasonable person standard sets the minimum of community
expectations, and those able to provide more are expected to do
so.

132
§ 3.04 The Child Standard of Care

Most jurisdictions hold children to a variation of a standard that


compares their conduct to other reasonable children of the
same age, experience, and intelligence under like circumstances.
While it is objective in that it compares the child to an external
standard of other children, it is far more subjective than the
adult reasonable person standard as it allows the jury to
consider the child's specific qualities such as experience and
intelligence.

133
[A] Adult Activities

Many jurisdictions have concluded that children should not be


entitled to special treatment when they are engaged in adult or
inherently dangerous activities, such as driving a car.

134
Example: Ten year-old Bonnie drives Father’s car through
Neighbor’s bay window. Is Bonnie Liable? What will the standard
of care be?

135
Negligence
Chapter 4
THE DETERMINATION OF UNREASONABLENESS: BREACH OF
DUTY, CUSTOM AND THE ROLE OF THE JURY

136
§ 4.01 Overview

Breach of duty is the defendant's failure to act as a reasonable


person would have under the same or similar circumstances.

More simply, breach is unreasonable conduct by the defendant.


Where reasonable minds can disagree, the jury is charged with
the task of deciding whether the defendant has breached a duty.

137
§ 4.02 The Risk Calculus

Negligence liability is imposed where the defendant engages in


unreasonable risk creation, situations where the defendant
creates risks that a reasonable person would not. This
determination of unreasonableness considers the risks that
should have been foreseen at the time of the defendant's
conduct, not through hindsight after the harm occurred.

How is unreasonableness determined? We look at whether the


burden of avoiding the harm is less than the probability of that
harm occurring multiplied by the likely seriousness of the harm if
it does occur. 138
[A] Probability

The probability factor seeks to measure the likelihood of the


harm-causing occurrence taking place.

Although probability must be considered in relation to


magnitude and burden, where there is a minuscule likelihood of
harm it is doubtful that the defendant breached a duty.

139
[B] Magnitude of the Loss

The magnitude of the loss looks at the likely harm flowing from
the injury-causing event when it occurs.

The proper focus is neither the most severe possible harm nor
the least severe; rather, it is what a reasonable person would
foresee as the likely harm.

140
[C] Burden of Avoidance

An analysis of burden requires consideration of such things as


the costs associated with avoiding the harm, alternatives and
their feasibility, the inconvenience to those involved and the
extent to which society values the relevant activity.

141
§ 4.03 The Jury Role

Where reasonable minds could disagree, the jury decides


whether the defendant acted unreasonably.

142
Negligence
Chapter 5
PROOF OF
BREACH

143
§ 5.01 Overview

The plaintiff has the burden to prove each element of a


negligence cause of action by a preponderance of the evidence.
If the plaintiff fails to carry this burden, the case must necessarily
be decided for the defendant. It is incumbent upon the plaintiff
to put on enough evidence so that a jury can find that more
likely than not the defendant failed to act reasonably.
The happening of an accident is never enough by itself to permit
a jury to find that a defendant has behaved unreasonably.

144
§ 5.02 Kinds of Evidence

There are two key forms of evidence that a plaintiff can use in
attempting to establish negligence by the defendant: direct and
circumstantial.
Direct evidence is evidence that comes from personal knowledge
or observation.
Circumstantial evidence is proof that requires the drawing of an
inference from other facts.
So long as the jury can draw a reasonable inference (as opposed
to speculate) the circumstantial evidence will be admitted.

145
§ 5.03 Res Ipsa Loquitur

Res ipsa loquitur, an important form of circumstantial evidence,


may be relevant to a plaintiff's efforts to establish the
defendant's unreasonable conduct.
In most negligence cases, the plaintiff specifies what the
defendant allegedly did unreasonably.
Res ipsa loquitur is most important and has its greatest impact in
cases where the plaintiff is unable to make specific allegations
about what the defendant did wrong.

146
§ 5.03 Res Ipsa Loquitur (continued)

The conditions traditionally required for the application of res


ipsa loquitur are:
“an accident that normally does not happen without negligence;
exclusive control of the instrumentality by the defendant; and
absence of voluntary action or contribution by the plaintiff.”

In order for the plaintiff to have the benefit of res ipsa loquitur,
she must convince the jury that each of these factors more likely
than not exists.

147
Example: Pedestrian is hit on the head by a falling flour barrel.
He sues the company that manufactures flour. The judge could
say “tell me what fault the company has committed so as to
establish negligence”. The pedestrian has no idea! He was
walking down the street minding his own business. Here, the
courts asked “Do flour barrels normally fall from buildings in the
absence of negligence”?

148
[B] Probably Negligence

A plaintiff must persuade a jury that more likely than not the
harm-causing event does not occur in the absence of negligence.
The plaintiff does not have to eliminate all other possible causes
for the harm, nor does the fact that the defendant raises
possible non-negligent causes defeat plaintiff's effort to invoke
res ipsa loquitur. The key is that a reasonable jury must be able
to find the likely cause was negligence.

149
Example: Driver hits Felix, who is sitting on a park bench on a
clear afternoon. Felix does not see anything as the car hits him
from the back. Here we have a desperate plaintiff who does not
know what happened.

(1) Does this normally happen in the absence of negligence?

(2) Was any negligence attributable to the Driver?

150
[C] Probably the Defendant

A plaintiff seeking to rely on res ipsa loquitur must connect the


defendant to the harm. The Plaintiff must usually prove
defendant had exclusive control over what caused the harm.

151
Negligence
Chapter 6
STATUTORY STANDARDS OF CARE –
“NEGLIGENCE PER SE”

152
§ 6.01 Overview

The “negligence-per-se” doctrine provides that in certain


situations a criminal statute (or administrative regulation or
municipal ordinance) may be used to set the standard of care in
a negligence case.
These statutes make no mention of civil liability but, rather,
impose fines or even imprisonment as punishment for those
violating their dictates.
Where appropriate, a specific legislative standard replaces the
more general reasonable person standard.

153
§ 6.02 Factors Used for Determining the Propriety of Adopting a
Statute As the Standard of Care

To decide whether a statute should be adopted as the standard


of care in a negligence case, a judge must determine that the
statute provides the sort of specific guidance that justifies its use
by a civil court.
Further, a judge must examine the statute in order to determine
whether the statute was designed to protect against the type of
harm suffered by the plaintiff and whether the class of persons
designed to be protected by the statute includes the plaintiff.

154
[A] Type of Harm

It is easy to determine the type of harm against which a statute


was designed to protect when the statutory purpose can be
discerned readily from the language of the statute or from its
clear legislative history.
Often the statutory purpose is not readily discernible from the
statute's language or legislative history, however.
Here judges have great discretion.

155
[B] Plaintiff in Protected Class

Sometimes it is easy for a judge to determine the class of


persons a statute was designed to protect, as where legislation is
passed to promote worker safety.

Sometimes, however, the scope of the protected class is


uncertain.

156
§ 6.03 Effects of Non-Adoption and of Adoption of Statute

[A] Effects of Non-adoption of a Statute

The judge's determination that the proffered statute should not


be adopted as the standard of care does not foreclose P's
recovery for negligence.

The case proceeds under the usual “reasonably prudent person”


standard of care.

157
[B] Effects of Adoption of the Statute and Statutory Violation

In most jurisdictions, the statute replaces the usual “reasonably


prudent person” standard of care.
To determine breach, all the jury needs to find is that the statute
was violated.
The plaintiff does not automatically recover upon a finding of breach of
the statutory standard of care because the plaintiff must still establish
the other elements of a negligence cause of action.
A minority of jurisdictions do not follow the negligence-per-se
approach.
In these jurisdictions, the standard of care remains that of a
reasonably prudent person and the relevant statute is simply
admitted for the jury's consideration in determining whether D 158
§ 6.04 The Role of Excuse

Modern courts have recognized excused statutory violations.


Acceptable excuses include: a sudden emergency not of the actor's
making; compliance would involve greater danger than violation; the
actor neither knows nor should know of the occasion for diligence;
the actor has some incapacity rendering the violation reasonable;
or, after reasonable efforts to comply, the party is unable to do so.
In most states, a judge makes an initial determination that the
proffered excuse is appropriate and the jury considers its
application. If there is an excused violation, the statute no longer
affects the
outcome.
159
§ 6.05 Negligence Per Se and Children

Most jurisdictions have concluded that the child standard of care


should apply even where the child has violated a statute.

Rather than using the statute as the standard of care, the


standard is that of a reasonable child of the same age, maturity,
intelligence and experience.

The statutory violation may be relevant to breach of duty.

160
Chapter 7
CAUSE-IN-
FACT
§ 7.01 Overview

It is generally accepted that tort liability is dependent on proof


that the defendant's culpable conduct or activity was the actual
cause of the plaintiff's injury.

161
§ 7.02 “But For” Analysis

The traditional and still dominant test for actual causation is the
“but for” test.

For the defendant to be held liable, the plaintiff must establish that
but for the defendant's culpable conduct or activity the plaintiff
would not have been injured.

162
Example:

A statute requires all vessels to have


lifeboats. D sends out a boat without
lifeboats.
P, a sailor, falls overboard in a storm so heavy that, even had
there been a lifeboat, it could not have been launched.
P drowns.
Even assuming that P was negligent per se, D’s failure to provide
lifeboats is not a cause in fact of P’s death, because that death
would have occurred even without the failure.
Therefore, D is not liable.
163
Example: Charter Vessel Owner is negligent for failing to keep life
preservers on board. Man falls off Vessel into shark’s waiting
jaws. Is the failure to have life preservers the cause in fact of
Man’s death?

164
§ 7.03 Substantial Factor Test / Concurrent causes

The “substantial factor” test requires that the defendant


materially contributed to the plaintiff's injury.
Sometimes D’s conduct can meet the “cause in fact”
requirement even though it is not a “but for” cause. This
happens when two events concur to cause harm, and either one
would have been sufficient to cause substantially the same harm
without the other.
Each of these concurring events is deemed a cause in fact of the
injury, since it would have been sufficient to bring about the
injury.
165
Example:
Sparks from D’s locomotive start a forest fire; the fire merges
with some other unknown fire, and the combine fires burn P’s
property.
Either fire alone would be sufficient to burn P’s property.
Therefore, D’s fire is a cause in fact of P’s damage, even though it
is not a “but for” cause.

166
Example: Al and Bill negligently light fires. The fires burn
separately for a time, but ultimately combine to burn down
Plaintiff’s house. “But for” won’t work. Who is liable?

167
§ 7.04 Proof Problems in Cause-in-Fact

[A] Shifting the Burden of Proving Causation

Where a small number of defendants have engaged in


substantially simultaneous culpable conduct imposing similar
risks on the victim, most courts will shift the burden of proof by
requiring defendants to prove they were not the actual cause.

168
Example: Moe, Larry and Curly are out hunting quail. Moe and
Larry shoot ... one piece of bird shot ends up in Curly’s eye. No
one can tell which shot hit Curly. “But for” won’t work, because
one shot curly, and the other did not… we just don’t know who
did what. Who is liable?

169
Negligence
Chapter 8
Proximate or
Legal cause

170
§ 8.01 Overview

The plaintiff must prove the defendant's culpable conduct is the


proximate cause of the plaintiff's injuries.
“Proximate” or “legal” cause adds to the requirement that the
defendant's culpable conduct be the actual cause of the
plaintiff's injury and will preclude recovery when the causal
relationship between the defendant's conduct and the plaintiff's
injury does not justify imposing tort responsibility on the
defendant.

171
§ 8.01 Overview (continued)

The proximate cause requirement is a policy determination that


a defendant, even one who has behaved negligently, should not
automatically be liable for all the consequences, no matter how
far-reaching and improbable, of his act.

Today, the proximate cause requirement usually means that D


will not be liable for the consequences that are very
unforeseeable.

172
§ 8.02 Proximate Cause Tests

[A] Foreseeability Test – Definition

The leading test for proximate cause focuses on whether the


defendant should have reasonably foreseen, as a risk of her
conduct, the general consequences or type of harm suffered by
the plaintiff.
In essence, the foreseeable harm test requires (1) a reasonably
foreseeable result or type of harm, and (2) no superseding
intervening force. The extent and the precise manner in which
the harm occurs need not be foreseeable.
173
Example:
D, driving carelessly, collides with a car driven by X.
Unbeknownst to D, the car contains dynamite which explodes.
Ten blocks away, a nurse who was carrying P, an infant, is startled
by the explosion, and drops P.
P will not be able to recover against D, because the episode was
so far-fetched – it was so unforeseeable that the injury would
occur from D’s negligence – that courts will hold that D’s careless
driving was not the “proximate cause of P’s injuries.

174
An intervening force is a new force which joins with the
defendant's conduct to cause the plaintiff's injury.
It is considered intervening because it has occurred sequentially
in time after the defendant's conduct.
If the intervening force is characterized as superseding,
proximate cause is not established even though the type of harm
is foreseeable.
An intervening force is generally characterized as superseding
only when its occurrence appears extraordinary under the
circumstances.
Elements to consider for proximity:
Breach  (time)  (actions)  (distance)  Injury 175
[B] “Practical Politics” and “Rough Sense of Justice” Test

In a famous dissent to Chief Justice Cardozo's opinion in the


renowned Palsgraf v. Long Island R.R. (N.Y. 1928), decision,
Justice Andrews considered the appropriate tests for proximate
cause.

Ultimately he concluded that proximate cause is a question of


public policy, fairness and justice, which cannot be reduced to
any mechanical formula.

176
Indirect Cause Cases fact pattern (further ripples)

Arises when an affirmative intervening force comes into motion


after D's negligent act and combines with it to cause P's injury
(here we must look at the intervening force and the result)

177
i. Foreseeable results/foreseeable intervening forces — D
ALWAYS liable

Example: D leaves his car keys in the ignition, and the car is
unlocked, while going into the store to do an errand. X comes
along, steals the car, and while driving fast to get out of the
neighborhood, runs over P. If the court believes that the risk of
theft is one of the things that makes leaving one’s keys in the
ignition negligent, the court will almost certainly conclude that
X’s intervening act was not superseding so as to exclude liability
of D.

178
Example:
D is a tavern owner, who serves too much liquor to X, knowing
that X arrived alone by car.
D also does not object when X gets out his car keys and leaves.
If X drunkenly runs over P, a court will probably hold that X’s
conduct in negligently (drunkenly) driving, although intervening,
was sufficiently foreseeable that it should not absolve D of
liability.

179
• Intervening (subsequent) medical malpractice — always
foreseeable

Example: Negligent Driver hits Pedestrian and breaks his leg.


Pedestrian is rushed to hospital where a doctor sets his leg
improperly, causing Pedestrian to develop an infection. Is
Driver’s negligence the proximate cause of Pedestrian’s
infection?

180
• Intervening negligent rescue — always foreseeable

Example: Negligent Driver hits Pedestrian and breaks his leg.


Rescuer attempts to drag Pedestrian out of harm’s way, but
instead drags Pedestrian into the path of another vehicle,
causing Pedestrian further harm. Is Driver’s negligence the
proximate cause of Pedestrian’s further harm?

181
• Reactions to D's conduct (efforts to protect persons or
property) – always foreseeable

Example: D, driving negligently, sideswipes P’s car on the


highway. P panics, thrusts the wheel to the right, and slams into
the railing. Even though most drivers in P’s position might not
have reacted in such an extreme or unhelpful manner, P’s
response is not sufficiently bizarre to constitute a superseding
cause.

182
Example: Defendant’s negligence causes crowd to flee from the
scene. A fleeing crowd member knocks over Plaintiff, breaking
his arm. Is Defendant’s negligence the proximate cause of
Plaintiff’s broken arm?

183
• Subsequent accidents caused by original injury – always
foreseeable

Example: Negligent Driver hits Pedestrian and breaks his leg.


Pedestrian is rushed to hospital where his leg is set properly.
That night, Pedestrian slips off his crutches while climbing stairs
and breaks his arm. Is Driver’s negligence the proximate cause of
Pedestrian’s broken arm?

184
ii. Unforeseeable results/foreseeable intervening forces -
Example: Negligence of a Third Party — D NOT liable

Example: Defendant negligently spills coffee on the sidewalk,


causing the Pedestrian to walk in the street. Speeding Motorist
hits Pedestrian. Is Defendant’s negligence the proximate cause of
Pedestrian’s injuries?

185
Example: Defendant negligently blocks sidewalk, forcing
Pedestrian to walk in the street. Speeding Motorist hits
Pedestrian. Is Defendant’s negligence the proximate cause of
Pedestrian’s injuries?

186
iii. Unforeseeable results/unforeseeable intervening forces –
Example: Acts of God — D NOT liable

Example: Assume that it is negligent to one’s neighbors to build


a large wood pile in one’s back yard, because this may attract
termites which will then spread. D builds a large woodpile. An
unprecedentedly strong hurricane sweeps through, takes one of
the logs, and blows it into P’s bedroom, killing him. The
hurricane will probably be held to be a superseding intervening
cause, because it was so strong as to be virtually unforeseeable,
and the type of harm it produced was not of the type that made
D’s conduct negligent in the first place.
187
Example: Workman leaves materials on roof despite predictions
of a windstorm. Windstorm arrives and blows materials off roof.
Falling materials hit Pedestrian. Is Workman’s failure to remove
materials the proximate cause of Pedestrian’s injuries?

188
[C] “Egg-shell” Plaintiff Personal Injury Rule

Rule—For the purpose of assessing damages, D takes P as he


finds him – no need to foresee the extent of the injuries. D must
pay full extent of damages suffered by P, even if surprising in
scope.

The defendant is liable even if the victim suffers physical injury


far more severe (e.g., heart attack) than the ordinary person
would be anticipated to have suffered from the accident.

189
Example: Bob cuts person at cocktail party. Unfortunately, that
person suffers from hemophilia, leading to his death. Liability for
all consequent damages?

190
Negligence
Chapter 9
Duty in
negligence cases

191
§ 9.01 Overview

The element of duty establishes that there is a legally recognized


relationship between the defendant and the plaintiff that
obligates the defendant to act (or to refrain from acting) in a
certain manner toward the plaintiff. Whether a duty exists is
often a policy-based determination and, thus, it is left to a judge
to make the determination of whether a duty exists. The duty
concept has been expanding to the point that now one engaged
in risk-creating conduct generally owes a duty to avoid causing
foreseeable personal injuries to foreseeable plaintiffs.

192
§ 9.02 Nonfeasance

Typically, there is no duty owed in a nonfeasance context.


Nonfeasance is generally the failure to intervene to confer a
benefit upon another. Misfeasance often consists of affirmative
acts of risk-creating conduct, doing something that a
reasonable person would not do.

Misfeasance can also be shown by a negligent omission – failing


to do something that a reasonable person would have done.
Either risk-creating affirmative acts or negligent omissions
generally lead to the finding of a duty.
193
Typically, nonfeasance-based actions arise where the plaintiff
contends that the defendant should have intervened to rescue
the plaintiff, or where the claim is that the defendant should
have prevented harm to the plaintiff by controlling a third party
or by taking measures to protect the plaintiff from injury.
Courts will find a duty in these contexts only in limited
situations.

194
[A] Duty to Rescue

The clear general rule remains that a person does not have a
duty to aid another. Courts consistently have refused to require a
stranger to render assistance, even where the person could have
rendered aid with little risk or effort. There are a variety of
reasons given for the law's no-duty-to-rescue rule.

The no-duty-to-rescue rule, nonetheless, has been powerfully


criticized as devaluing human life and celebrating selfishness.
Discomfort with the rule has led courts to fashion
various exceptions.
195
Example: D, passing by, sees P drowning in a pond. D could easily
pull P to safety without risk to D, but instead, D walks on by. Is D
liable?

196
[1] Creating the Peril

A well-established exception to the no-duty-to-rescue rule


applies when the need for rescue arises because of the
defendant's negligence. Also, some jurisdictions have found an
exception where a person's fault-free conduct gives rise to the
need to rescue. Indeed, there is movement toward imposing
rescue obligations on those who are connected in any way to the
need for rescue.

197
Example: A car driven by D strikes P, a pedestrian. If D is found to
have been driving completely non-negligently, and the accident
is due to P’s carelessness in crossing the street, does D have a
duty to stop and give reasonable assistance?

198
[2] Special Relationships

Courts have imposed a duty to rescue when justified by a


“special relationship” between the parties such as a common
carrier-passenger, innkeeper-guest and ship captain-seaman.
There seems to be a movement to find a duty to aid or protect in
any relation of dependence or of mutual dependence.”

Example: P gets his finger stuck in an escalator operated by D, a


store where P is a customer. If D does not give P assistance, will
D be liable?

199
[3] Undertaking to Act
While people generally have no obligation to intervene, once
they do, a duty arises. There are different views about the extent
of the obligation: under the traditional view, once a person
undertakes to rescue, he must not leave the victim in a worse
position; under the more modern view, the rescuer is obligated
to act reasonably once he has begun to act.

D is especially likely to be found liable if he begins to render


assistance, and this has the effect of dissuading others from
helping P.

200
Example: If D stops by the roadside to help P, an injured
pedestrian, and other passers-by decline to help because they
think the problem is taken care of, may D abandon the attempt
to help P?

201
[4] Contract

Occasionally, a rescue obligation arises from contract. There is


debate about the extent to which a defendant's gratuitous
promise, without more, gives rise to a duty.

202
[B] Duty to Control and Protect

A person typically is not legally obligated to control the conduct


of another or to take steps to protect another from harm.

203
[1] Control

While generally a person has no obligation to control another


person's conduct to prevent harm to a third person, exceptions
arise where there is a special relationship.

The relationships giving rise to a duty to control require some


relationship between the defendant and the third party,
combined with knowledge (actual or constructive) of the need
for control. There can be substantial debate about which
relationships give rise to a duty to control.

204
Example: D, a storekeeper, fails to take action when X, obviously
a deranged man, comes into the store wielding a knife. P, a
patron, is stabbed. Is D liable?

205
Negligence
Chapter
10
LAND OCCUPIER
DUTY

206
§ 10.01 Overview

Under the common law approach, the measure of the duty owed
depends on the status of the person entering the land – whether
the entrant is a “trespasser,” a “licensee,” or an “invitee.” The
status of the person entering the land determines the standard
of care owed by the land occupier. Some jurisdictions have
rejected the status approach to liability, using a generalized duty
of ordinary care instead.

207
§ 10.02 The Common Law Status Approach

The common law approach to landowner liability measures the


duty owed by a land occupier to persons entering the property
by the status of the entrant. Because of the value attached to
private land ownership, the law developed in a way that was
highly protective of these interests.

208
[A] Trespassers

A “trespasser” is one who enters or remains on the property in


the possession of another without the permission (express or
implied) of the land occupier. The duty owed to trespassers was
(and in many ways remains) extremely limited.
The only obligation initially imposed on land possessors was to
refrain from willfully harming the trespasser. Courts expanded
the duty owed to trespassers to include requiring warnings
about “traps.” At the outset, the concept of “traps” was narrowly
defined. More recently, courts have engaged in some creativity,
adopting a broader interpretation of “trap.”
209
[1] Frequent or Known Trespassers

In most common law jurisdictions, the traditional rule has been


altered in the case of known or frequent trespassers.

Where a land occupier actually is aware of the presence of a


trespasser and knows that the trespasser is approaching a non-
evident artificial (human made) condition, the land occupier is
obligated to warn the trespasser if there is danger of serious
bodily harm or death.

210
Also, if the land occupier is on notice of frequent trespassing, or
has reason to know of such, an obligation to warn of hidden
dangers known to the land possessor and risking serious injury
or death may be imposed.

No warning need be given of conditions on the land that a


trespasser would be expected to discover or which are inherent
in the use of the land.

Further, the land occupier who knows of a trespasser's presence


must use reasonable care for the protection of the trespasser in
carrying on activities.
211
[2] Child Trespassers

The rules barring recovery for the majority of injured trespassing


children caused discomfort for the courts. By the 1870s, courts
began broadening the land possessor's duty to trespassing
children in limited situations.
This more lenient approach has become known as the “attractive
nuisance doctrine.”
Under the Restatement a child trespasser will be owed a duty of
ordinary care if a judge balances several factors and finds that
they support providing the child trespasser special treatment.

212
• Infant Trespassers—"Attractive Nuisance Doctrine"

Owner must exercise reasonable care to protect against


reasonably foreseeable harm to children caused by artificial
conditions
P must show:
a) Dangerous condition
b) About which owner knew or should have known
c) Owner knows or should know children frequent the
area
d) Condition is likely to cause injury (i.e., dangerous because of
child's inability to appreciate risk)
e) Expense of remedying situation is slight compared to
magnitude of risk 213
[B] Licensees

A licensee is someone who enters the land with the express or


implied consent of the land possessor, as is the case with social
guests.
The licensee takes the property in the condition in which the
land possessor uses it.
A land possessor may be liable to a licensee injured by a
condition on the property where the land possessor knows of a
dangerous condition on the property, fails to make the condition
safe or to warn the licensee about the risk involved, and the
licensee does not know about the danger nor would be expected
to discover the dangerous condition. 214
[C] Invitees

There are two primary types of invitees: business invitees and


public invitees.
Business invitees are on the premises for the potential financial
benefit of the land occupier.
Public invitees are on land held open to the public at large.
As to invitees, land possessors must use reasonable care in
maintaining the premises and in their activities. This often
entails taking affirmative steps to discover dangers on the
property. The obligation of the land possessor to an invitee then
is one of reasonable care.
215
§ 10.03 Land Possessor Duty to Those Outside the Land

The common law rule provides that a land possessor owes no


duty to those outside the land for natural conditions on the land.
Where harm is occasioned by an artificial condition or the land
possessor's activity, however, a duty is recognized.
Some jurisdictions have rejected the no-duty rule regarding
natural conditions and have replaced it with a generalized duty
of reasonable care.

216
§ 10.04 Quick Reference

• General Points

With respect to activities being conducted on the land, owner


generally has a duty to exercise reasonable care under the
circumstances (BUT no duty is ever owed to undiscovered
trespassers)

217
• Owner is NEVER liable for open or obvious conditions—P
expected to care for himself

For conditions hidden or concealed from P, there are two ways


owner can satisfy his duty to P (either works):

a) Fix or repair the condition (i.e., eliminate the problem), OR


b) Give a warning—unknown hazard becomes a known
hazard

• Undiscovered Trespassers—NO duty owed (b/c


unforeseeable) 218
• Discovered/Anticipated Trespassers (awareness)

Must protect against “man made death traps”:


a) Concealed or hidden conditions—(remember: no duty to
protect against obvious conditions)
b) Artificial conditions—there is no duty to protect against
natural conditions
c) Conditions known to landowner
d) Conditions involving risk of death or serious bodily harm—
condition must be highly dangerous (no duty for minor
harms)
219
Example: I own a piece of rural property. In the summer,
possums are foraging on my land. I decide to set some steel traps
with sharp teeth to try and catch the possums. As autumn
arrives, I leave the traps there, and leaves start falling. The leaves
cover the traps, so as they are now concealed. Also, in the
autumn, hikers usually come on my property, although I have
never given them permission. One weekend, one of the hikers
steps on the trap and injures her foot. She sues for negligence.
Did I owe her a duty of care?

220
Chapter 11
DUTY LIMITED BY KIND OF HARM

221
§ 11.01 Negligent Infliction of Emotional Distress

[A] Overview

Historically, tort law provided compensation for a victim's mental


distress only when it followed physical injury.
Recovery for this emotional upset, parasitic to the plaintiff's
claim for physical harm, is typically known as “pain and
suffering.”
Now, in certain limited circumstances, negligently inflicted
mental distress that does not follow from physical harm is
recognized as a basis for recovery.
222
Traditionally, as a prerequisite to recovery for mental distress,
the defendant's negligence must have caused some form of
physical impact on the plaintiff's person. Most states today only
require that the plaintiff have been in risk of physical impact,
sometimes referred to as being within the “zone of impact” or
the “zone of danger.” – also known as the “near miss” theory.

Most states also require that the victim's mental distress be


sufficiently severe to cause physical symptoms of the distress.
Some jurisdictions have flirted with a much broader recovery for
pure emotional distress – dispensing with a requirement of
physical manifestations and broadly defining the class of proper
plaintiffs. 223
A separate development has been the gradual recognition of
bystander recovery for negligently inflicted emotional distress.
A majority of states allow a bystander to recover only if the
bystander is also within the zone of physical risk.

A significant minority of states now allow recovery for


bystanders who are not in risk of physical impact if they (1) are
physically near the accident; (2) have contemporaneous sensory
perception of the accident; and (3) are closely related to the
victim.
Most of the states following this approach also continue to
require that the bystander-plaintiff suffer some physical
manifestation of her distress. 224
B. Elements (three):
1 Act by D
2Which exposes P to a foreseeable risk of physical injury or
threat of impact
Majority rule—requires "zone of danger" (i.e., "near miss")
Modern Trend ("Bystander Theory")—allows recovery if:
P and injured party were closely related (i.e., family
members)
P was present at the scene
P observed the injury

225
[3] Some subsequent physical manifestation (e.g., nervous
breakdown, miscarriage)

This is not required for erroneous report of relative's death or


mishandling of corpse, or a doctor negligently telling a patient
they are suffering from a terminal disease when they are in fact
not.

In these cases, there is no need to prove subsequent physical


manifestation.

226
Example: Paul is standing next to two pedestrians who are hit by
Defendant’s negligently driven car. Paul suffers from severe
emotional distress. He also breaks out in hives and has heart
palpitations. Does Defendant owe Paul a duty of care?

227
Example: Paula is watching from her window when two
pedestrians are hit by Defendant’s negligently driven car. Paula
suffers from severe emotional distress. She also breaks out in
hives and has heart palpitations. Does Defendant owe Paula a
duty of care?

228
Example: Paula suffers emotional distress after seeing her child
hit by Defendant’s negligently driven car as they were walking
hand in hand back from school. She also breaks out in hives.
Does Defendant owe Paula a duty of care?

229
Chapter 12
Damages

230
§ 12.01 Overview

Damages in torts constitute the “money awarded to the person


injured by the tort of another.” Tort damages include nominal
damages, compensatory damages, and punitive damages.

Nominal damages are a symbolic award (often one dollar) given


to the plaintiff when liability for a tort is established but no
actual harm is proven.

231
Compensatory damages are awarded to a person to indemnify
for personal injury, property, and other economic harm
sustained by the victim. Compensatory damages are awarded for
both pecuniary and non-pecuniary losses. Unlike economic loss,
pain and suffering, and other forms of mental distress have no
obvious monetary equivalent. This valuation problem has
generated controversy over the desirability of compensating for
pain and suffering at all.

Punitive damages are awarded to punish and deter particularly


egregious conduct.

232
§ 12.02 Property Damages

Damages for permanent deprivation or destruction of property


are generally measured by the market value of the property at
the time of the tort. If real or personal property is damaged but
not destroyed, courts generally compensate the victim for the
diminished market value of the property but sometimes award
the cost of repairs instead of diminished value.

233
§ 12.03 Personal Injury

Personal injury victims under tort law can be compensated for


(1) medical expenses; (2) lost wages or impaired earning
capacity; (3) other incidental economic consequences caused by
the injury; and (4) pain and suffering.

234
§ 12.04 Mitigation or the Doctrine of Avoidable Consequences

Injured victims have a responsibility to act reasonably to limit or


“mitigate” losses incurred. If a plaintiff fails to act reasonably to
mitigate injuries, the defendant will not be held liable for
incremental losses that otherwise could have been avoided.

235
§ 12.05 Punitive Damages

Punitive damages are discretionary and awarded when a tort is


committed with malice. The United States Supreme Court has
held that punitive damages must bear some relationship to
potential harm. Also, many states limit punitive damages awards.

236
Elements of willful or reckless disregard of plaintiff’s rights:

• The existence and magnitude of the product danger to the public;


• The cost or feasibility of reducing the danger to an acceptable level;
• The manufacturer’s awareness of the danger, the magnitude of
the danger, and the availability of a feasible remedy;
• The nature and duration of, and the reasons for, the manufacturer’s
failure to act appropriately to discover or reduce the danger;
• The extent to which the manufacturer purposefully created the danger;
• The extent to which the defendants are subject to federal
safety regulation;
• The probability that compensatory damages might be awarded against
the defendants in other cases; and
• The amount of time which has passed since the actions sought to
be deterred.
237
§ 12.06 Collateral Source Rule

Under traditional common law doctrine, the plaintiff's recovery


against the defendant is not affected by compensation the
plaintiff received for the loss from other sources such as
insurance. Such collateral sources for recovery are not disclosed
to the jury under the collateral source rule.

Numerous reform statutes, most notably in the context of


medical malpractice, now reject the collateral source rule and
allow the jury to consider such insurance payouts and deduct
them from the defendant's liability.
238
Chapter 13
JOINT & SEVERAL LIABILITY

239
Overview Definition

§ 13.01 Joint Tortfeasors

Joint tortfeasors are two or more individuals who either (1) act in
concert to commit a tort, (2) act independently but cause a
single indivisible tortious injury, or (3) share responsibility for a
tort because of vicarious liability.

240
Under traditional common law, each joint tortfeasor is “jointly
and severally” liable for the plaintiff's total damages.

This means that each individual is fully liable to the plaintiff for
the entire damage award.

If the plaintiff is unable to collect a co-tortfeasor's portion of the


liability, the tortfeasor(s) from whom the plaintiff can collect are
responsible for the other tortfeasor's (s') share.

241
Example: D1 negligently scratches P. P goes to the hospital,
where she is negligently treated by D2, a doctor, causing her to
lose her arm. P can recover her entire damages from D1, or her
entire damages from D2, though she cannot collect twice.

242
[A] Acting in Concert

“Acting in concert” is the tort equivalent of being a criminal


accessory or co-conspirator.

If an individual intentionally aids or encourages another to


commit a tort, he is as liable as the individual who
actually committed the physical acts of the tort.

243
[B] Vicarious Liability

A defendant may be jointly liable for the actions of another


through vicarious liability.
Vicarious liability automatically imposes tort responsibility on a
defendant because of his relationship with the wrongdoer.
The most frequent examples of vicarious liability are when
employers are held liable under a theory of respondeat superior
for the actions of employees within the scope of their
employment, and when parents are held liable for the torts of
their children.

244
Example: Employee injures P. P recovers against Employer on a
theory of respondeat superior. Employer will be entitled to
indemnity from Employee; that is, Employee will be required to
pay to Employer the full amount of any judgment that Employer
has paid.

245
§ 13.02 Special Problems After Comparative Fault

[A] Comparative Negligence (what most courts use today)

Traditionally, each liable defendant, if able, paid an equal pro


rata share of the damages to the plaintiff based on the number
of joint tortfeasors. This traditional approach has now been
replaced in many states by a system of comparative allocations
of responsibility among joint tortfeasors.
Under a comparative approach, instead of dividing liability
equally by the number of joint tortfeasors, liability is divided by
the proportion of responsibility each tortfeasor bears to the
plaintiff for his injury. 246
Example: A jury finds that P was not at fault at all, that D1 was at
fault 2/3 and D2 was at fault 1/3. P’s damages are $1 million. P
can recover the entire sum from either D. But if P recovers the
entire sum from D1, D1 may recover $333,333 from D2.

247
[B] Contribution

Under joint and several liability principles, the tortfeasor sued


must still bear full responsibility for the injury to the plaintiff.

With the exception of tortfeasors liable for intentional torts, a


defendant required to pay the plaintiff more than her share of
the damage judgment (under either a pro-rata or comparative
system) can now seek in most states appropriate contribution
from her co-tortfeasor.

248
Example: A court holds that D1 and D2 are jointly and severally
liable to P for $1 million. P collects the full $1 million form D1. In
most instances, D1 may recover $500,000 contribution from D2,
so that they will end up having each paid the same amount.

249
Chapter 14
Defenses

250
§ 14.01 Overview

Traditionally, there were only two defenses to negligence:


contributory negligence and assumption of risk. Both constituted
complete defenses and completely barred the plaintiff from
recovery. In all but a handful of states, contributory negligence
has been converted by statute or judicial ruling into comparative
negligence. Unlike contributory negligence, comparative
negligence need not be a complete bar to the plaintiff's recovery,
but acts only as partial bar resulting in a percentage deduction
from otherwise recoverable damages.

251
§ 14.02 Contributory Negligence

[A] Definition

Contributory negligence is “conduct on the part of the plaintiff


which falls below the standard of conduct to which he should
conform for his own protection, and which is a legally
contributing cause . . . in bringing about the plaintiff's harm.”

Contributory negligence is a complete defense to negligence.

252
(1)Knowing contributory negligence/implied assumption of
risk.

(a)One sees the risk.


(b)Unreasonably, voluntarily takes on the risk.

(2)Unknowing contributory negligence

(a)P doesn’t see the precise risk.


(b)P is just careless.

(3) Only with knowing contributory negligence can we 253


[B] Last Clear Chance Doctrine

The last clear chance doctrine instructs the court to ignore the
plaintiff's contributory negligence if the defendant's negligence
occurred after the plaintiff's contributory negligence.

Most jurisdictions reject the last clear chance doctrine when


replacing contributory negligence with comparative negligence.

254
§ 14.03 Comparative Negligence

In almost all states, contributory negligence has been replaced


by some form of comparative negligence, often called
comparative fault.
Under comparative negligence, “the conduct on the part of the
plaintiff which falls below the standard of conduct which he
should conform to for his own protection and which is a legally
contributing cause . . . in bringing about the plaintiff's harm” is
only a partial bar to the plaintiff's recovery. Comparative
negligence reduces the plaintiff's recovery by the percentage of
responsibility for the injury attributable to the plaintiff.
255
When the plaintiff is also contributing to his/her tort!

Types
• Pure Comparative Negligence – System under which the
Plaintiff will always recover something even if the Plaintiff was
the predominant or majority negligent actor in the case.
• Modified Comparative Negligence – The recovery by the
Plaintiff is reduced by a percentage threshold based on his
contribution of fault, up to a maximum of 49% or 50%
(Depending on the States), after which Plaintiff is barred from
recovery.

256
Application

Example: Jury finds Plaintiff 70% at fault, Defendant 30% at fault.

• Under pure comparative negligence, Plaintiff recovers 30% of


her damages.
• Under modified comparative negligence, Plaintiff recovers
nothing!

257
§ 14.04 Assumption of Risk

[A] Definition

Along with contributory negligence, assumption of risk has


traditionally existed as a complete defense to negligence.

There are thus three basic elements to the assumption of risk.


The plaintiff must (1) know a particular risk and (2) voluntarily (3)
assume it.

258
B. Classifications of Assumption of Risk

1 Express Versus Implied Assumption of Risk

Assumption of risk is generally divided into two types: express


and implied. Express assumption of risk exists when, by contract
or otherwise, a plaintiff explicitly agrees to accept a risk. Implied
assumption of risk exists when the plaintiff's voluntary exposure
to risk is derived merely from her behavior, and not from explicit
assent.

259
[2] Express Assumption of Risk

If an assumption of risk is characterized as express, it can be


invalidated if it is found contrary to public policy. Conversely,
courts are likely to uphold express assumption of risk when the
plaintiff's participation is clearly voluntary, such as the decision
to engage in risky recreational pursuits.

260
Elements:

• Parties may agree that there shall be no obligation to take


precautions and hence no liability for negligence.
• Bargaining must be free and open.
• Against public policy to permit exculpatory agreements as to
transactions involving the public interest, as for example with
regard to public utilities, common carriers, and innkeepers.
• Cannot override a safety statute.

261
[3] Implied Assumption of Risk

The modern trend is to allow implied assumption of risk to be


absorbed into comparative negligence. This allows the jury to
treat assumption of risk as a partial defense.

262
§ 14.05 Immunities

[A] Overview

An immunity protects a defendant from tort liability. Unlike a


defense, it is not dependent on the plaintiff's behavior, but on
the defendant's status or relationship to the plaintiff.

263
[B] Parent-Child Immunity

Parent-child immunity precludes tort actions between parents


and their non-adult children. Parent-child immunity still exists in
some form in many jurisdictions.

264
[C] Governmental Immunity

Governmental immunity protects the government from tort


liability. Under the common law, the immunities were complete
and prevented any tort suits against the government.
Many states and the federal government have passed detailed
statutes modifying the immunities in specific instances. One
general provision normally included allows immunity for
discretionary functions but not ministerial acts. Discretionary
functions are policy-making decisions. Ministerial acts constitute
government conduct which implements or executes policy
decisions.
265
Who is the only person in the US who has full civil immunity?

The President.

What do you think about that?

266
Chapter 15
DEFAMATIO
N

267
§ 15.01 Overview

The law of defamation is particularly intricate due to its unique


blend of common law and First Amendment principles.

The tort of defamation permits recovery for reputational harm,


which was considered a grave injury in socially stratified England.
Over time, a number of common law privileges have been
recognized, making a plaintiff's defamation case more
challenging, however.

268
§ 15.02 Common Law Defamation

At common law, defamation was a strict liability tort.

As such, a plaintiff could recover without proving any fault on


the part of the defendant.

Furthermore, the falsity of the allegedly defamatory statement


was presumed.

Finally, in most instances, damages were presumed.


269
Thus, in most common law defamation actions the plaintiff only
had to prove (1) a defamatory statement (2) about the plaintiff
(3) that was "published."

The defendant then had the opportunity to try to assert a


defense, such as the truth of the statement.

Thus, at common law, a defendant could quite unwittingly


defame another and be responsible for significant damages.

270
[A] Defamatory Statement

To be defamatory under the general common law rule, a


statement must hold the plaintiff up to scorn, ridicule, or
contempt.

The Restatement provides that a communication is defamatory if


it "tends so to harm the reputation of another as to lower him in
the estimation of the community or to deter third persons from
associating or dealing with him.”

271
A defamatory statement, is one that harms reputation by
injuring a person's general character or causing personal
disgrace.

A court determines as a matter of law whether any


interpretation of the communication could be construed as
defamatory, while it is for a jury to decide whether the
statement in the case before it is actually defamatory.

272
Example: Talk Show Host calls Dr. Greene a “buffoon”. Is this a
“defamatory statement?”

273
[1] Defamatory to Whom?

The plaintiff must show that a "substantial and respectable


minority" or a "right-thinking minority" would comprehend the
defamatory nature of the communication.

This group can be quite small. If the group that could interpret
the communication in a way that injures the plaintiff's reputation
is of a blatantly anti-social nature, courts may deny the plaintiff a
defamation action.

274
[2] Statements Not Facially Defamatory: Inducement and
Innuendo

Some statements are facially defamatory; nothing needs to be


added for a reader to fully understand the defamatory nature of
the statement.

Other times the defamatory impact can only be understood by


the addition of extrinsic information. In such situations the
plaintiff is obligated to plead the extra facts needed to make the
statement defamatory ("inducement") or to explain the
defamatory impact ("innuendo") if it is not obvious.
275
[B] Of and Concerning the Plaintiff

The plaintiff must show that the defamatory communication was


understood as referring to her.

If the plaintiff can show this, it is irrelevant that the defendant


did not intend for the statement to refer to the plaintiff.

276
Similarly, even if the defendant intended to create a fictional
character, a defamation action will lie where recipients of the
communication reasonably believe that the character is really
the plaintiff.

Where the plaintiff is not expressly named in the


communication, the plaintiff must plead "colloquium" to connect
herself to the defamatory statement.

277
[C] Publication and Republication

A plaintiff must establish that the defamatory communication


was published, meaning that it reached one person other than
the defamation plaintiff.

The plaintiff must show that either the defendant intended to


publish the information or was negligent in so doing.

Any repetition of a defamation is considered publication, even if


the republisher attributes the statement to the initial source.
278
Example: Talk Show Host confronts Dr. Greene quietly in his
office, but is overheard by Eavesdropper. Has Talk Show Host
“published” his statement to Eavesdropper?

279
[D] Damages

In most defamation cases, a plaintiff's reputational injury may be


presumed, permitting the plaintiff to recover compensation
without any proof beyond the defamatory nature of the
communication.

In the defamation context, such damages are called "general


damages." General damages provide compensation for the
emotional trauma and harm suffered by the plaintiff whose
reputation was besmirched.

280
There are situations, however, where the plaintiff must plead
and prove a specific type of loss, called "special damages," in
order to prevail.

Special damages are specific economic losses flowing from the


defamation. If the plaintiff proves these special damages, she
may then recover general damages.

The damages recoverable to a defamation plaintiff depend on


whether the defamatory communication is considered libel or
slander and, if slander, whether the defamation falls into a
category denominated "slander per se."
281
[1] The Libel/Slander Distinction

Slander is an oral utterance while libel is a more permanent


expression.

282
Example: Talk Show Host says that Dr. Greene has performed
unnecessary surgeries. Is this a “defamatory statement?”

283
[2] Slander and Slander Per Se

Where the defamation is characterized as slander, the plaintiff


generally must meet the substantial burden of pleading and
proving special damages.

Since early common law, however, certain slanderous statements


were deemed so horrible that reputational injury to plaintiffs
could be presumed even without any proof of special damages.

284
The four traditional slander per se categories that permit
presumed reputational damages absent special damage are: (1)
communications that directly call into question the plaintiff's
competence to perform adequately in her trade or profession;
(2) statements claiming the plaintiff has a current, loathsome
disease; (3) allegations of serious criminal misbehavior by the
plaintiff; (4) and, suggestions of a lack of chastity in a woman.

285
[E] Common Law Defenses

Because at common law a plaintiff could often establish a prima


facie case of defamation quite easily, the defendant often had to
look to the available defenses in order to avoid liability.

286
[1] Substantial Truth

At common law, the defamatory communication was presumed


false, and it was incumbent upon the defendant to establish
truth as a defense.

While the defendant had to show the accuracy and truth of the
statement in issue, she did not have to show the literal truth of
every aspect – substantial truth is the test.

287
[2] Absolute Privileges

There are a few contexts that rely so heavily on unfettered


discourse that the law provides immunity from defamation
liability.
These absolute privileges typically arise in governmental
proceedings involving judicial, legislative and executive
communications.
In the judicial context, statements made in court or in official
court papers are absolutely privileged as long as relevant to the
court proceeding.

288
A similar absolute privilege applies to legislators and high-level
executive officers.

An absolute privilege protects a defendant even if she knew the


statement was false or published it in order to harm the
plaintiff's reputation.

289
Example: During floor debate, Senator falsely says that
Constituent cheated the welfare system. Can Constituent
successfully maintain a defamation action?

290
[3] Qualified Privileges

Qualified (or conditional) privileges have developed due to the


recognition that there are certain interests which could be
seriously impaired by the common law's strict liability approach
to defamation.

Qualified privileges are based on the social utility of protecting


communications made in connection with the speaker's moral,
legal or social obligations.

291
These privileges can be lost in several ways: by failing to have an
honest belief that the statement was true; by failing to have an
objectively reasonable belief that the statement was true; or by
disclosing the information to more people than necessary (that
is, excessive publication).

Another important qualified privilege in many jurisdictions is the


"fair and accurate report" privilege.

This privilege permits a report of public meetings, and probably


information in public records, provided that the report is an
accurate and unbiased account.
292
Chapter 16
INVASION OF PRIVACY

293
§ 16.01 Overview

The Restatement has defined four separate torts for invasion of


privacy: intrusion upon seclusion, appropriation of plaintiff's
name or picture, placing the plaintiff in a false light before the
public, and public disclosure of private facts. All four privacy torts
enjoy general judicial acceptance today.

294
§ 16.02 Intrusion Upon Seclusion

The Restatement defines the tort as: “One who intentionally


intrudes, physically or otherwise, upon the solitude or seclusion
of another or his private affairs or concerns, is subject to liability
to the other for invasion of his privacy, if the intrusion would be
highly offensive to a reasonable person.”

295
The tort addresses acts of intrusion and other interferences with
a victim's “zone of privacy.” There is no requirement that
information be obtained or communicated; it is the intrusion
itself that constitutes the interference. While such interference
may also constitute trespass, there is no requirement that
trespass be committed.

296
Example: Celebrity is swarmed by photographers on a public
street. Intrusion?

297
§ 16.03 Appropriation of Name or Picture and the Right of
Publicity

The Restatement defines the tort as: “One who appropriates to


his own use or benefit the name or likeness of another is subject
to liability to the other for invasion of his privacy.”

The tort applies to an unauthorized endorsement of a product. It


does not, however, apply to journalistic articles or books about a
person.

298
Example: Chocapic puts Celebrity’s picture on a cereal box.
Appropriation?

299
§ 16.04 False Light

The privacy tort of false light overlaps in large measure with


defamation.

The elements of false light which must be established by all


plaintiffs include the defendant's (1) publicizing (2) false facts (3)
that a reasonable person would object to. In the case of a
matter of public interest, all plaintiffs (public or private) must in
addition establish that the defendant acted with New York Times
malice (knowledge of falsity or reckless disregard toward the
truth).
300
Example: Newspaper publishes false story proclaiming Bob a
hero for rescuing ten children from a burning building. False
light?

301
§ 16.05 Public Disclosure of Private Facts

The tort elements include (1) publicity of (2) private facts (3)
highly offensive to a reasonable person which are (4) not of a
legitimate public interest.

The requirement that the facts be private has been rigorously


enforced by courts. For example, the United States Supreme
Court has held in two cases that information even
unintentionally placed in the public record and thus subject to
inspection cannot be actionable under the First Amendment.

302
English for
Law
Practice Questions

303
At a little league game, a seven-year-old boy was called out on strikes. The
boy’s father was so infuriated with the umpire’s decision that he shouted in
a loud voice, “Kill the umpire.” The boy, who was still holding his bat,
swung the bat at the umpire. The umpire ducked and the bat flew out of
the boy’s hands and struck a spectator, who was seriously injured.
In a tort action by the umpire against the boy:

(A) The umpire could recover only on an assault theory.


(B)The umpire could recover either on an assault theory or a negligence
theory.
(C) The umpire could recover only on a negligence theory.
(D) The umpire could not recover.

304
305
A physician performed scheduled surgery on her patient’s right ear for a
condition caused by prolonged and repeated infections in that ear. During the
surgery, the physician determined that her patient had been particularly
susceptible to this condition due to a previously unsuspected anatomical
abnormality. The physician reasonably believed that this same abnormality was
likely to exist in the patient’s left ear. Though the patient had not had many
infections in the left ear, if a similar course of recurring infections were to
transpire involving that ear, it would probably develop the same condition as
the right and require surgery. The physician therefore decided to perform
surgery on her patient’s left ear, although she had received his consent only to
operate on the right ear. The surgery was performed with due care and was
successful.
In an action by the patient against the physician:
(A)The patient will not recover because the extension of the operation was
successful.
(B)The patient will not recover because the extension of the operation was
carried out with due care.
(C)The patient will recover at least nominal damages on a negligence theory.
(D) The patient will recover at least nominal damages on a battery theory.
306
307
Two men stopped to have a drink together at a bar, where five other people
were also gathered. The two men had a few drinks and began to argue, hurling
insults at each other. After the first man threw a punch, a fight ensued, and the
second man was struck in the back of the head with a heavy object from
behind. He never saw who had struck the blow and he was knocked
unconscious. He was hospitalized for a severe concussion and still suffers from
severe headaches.
The injured man sued the first man as well as the five other persons present in
the bar at the time he was struck, seeking to recover damages for the blow to
his head. He presented the evidence above.
The injured man will:

(A)Prevail, because he was not the aggressor.


(B)Prevail, because the first man acted unreasonably.
(C)Prevail, because of the doctrine of res ipsa loquitur.
(D)Not prevail, because he cannot prove that there was concerted action against
him.
308
309
A man and a woman who were fierce business competitors were both
competing for a large job. The man submitted his bid and then went to the
woman’s office and told her, “If you leave this office, I’m going to get you!” The
woman merely laughed and said, “I’m about finished with my bid and will be
leaving in a few minutes.” The man left the office but placed a large, heavy
couch across the entrance to the woman’s office, hoping to keep her from
leaving. Meanwhile, the woman finished the bid and tried to leave her office,
but found that she could not open the door. She pushed against the door as
hard as she could and was eventually able to force it open, then ran all the
way to the place where bids were being taken and got her bid in with one
minute to spare. As usual, her bid was slightly lower than the man’s, and she
was awarded the contract.
If the woman sues the man, she has a cause of action for:
(A) Assault, but not false imprisonment.
(B) False imprisonment, but not assault.
(C) Both assault and false imprisonment.
(D) Neither assault nor false imprisonment.
310
311
A college student was holding a loud party at her house. The next door
neighbor was getting increasingly angry with the noise, and several
complaints to the local police station brought no results. The neighbor
called the student on the phone and told her that if she did not stop the
noise, he would “come over there and cut your throat.” The visibly shaken
student told her guests what had just happened, and they all decided to
leave immediately. The student was unable to sleep that night and
thereafter purchased an alarm system for her house and a gun that she
kept next to her bed.
If the student brings an action for intentional infliction of emotional distress
and succeeds, it will be because:
(A)The neighbor had the apparent present ability to make good on his
threat.
(B)The student suffered some physical harm as a result of her distress.
(C)The student suffered pecuniary injury as a result of his threat.
(D) The neighbor’s conduct was extreme and outrageous.
312
313
While returning from transporting a group of children to summer camp, a bus
driver and his assistant were caught in the leading edge of a forest fire raging
down the high mountains. Hurrying ahead of the flames and smoke, the driver
reached the last half-mile of a dirt road that ran to the main highway and safety,
but he discovered that the road ahead was already blocked by fallen, burning
foliage. Separating the driver’s bus from the main highway, which angled off to
the right, was the fenced property of a rancher. The bus driver drove across the
property to reach the main highway, damaging some turf and a fence, and
proceeded to the city.
If the rancher asserts a claim against the bus driver to recover for the damage to
his property, the rancher will probably:
(A)Lose, because the bus driver was acting to protect the lives of himself and his
assistant.
(B)Lose, because the bus driver acted as would any reasonably prudent person
under the circumstances.
(C)Win, because the bus driver damaged the rancher’s property when he drove
through the fence to get to the main highway.
(D)Win, because the bus driver intentionally drove across the property, knowing
it would cause damage.
314
315
A teenager who was totally blind in one eye and had only 10% vision in the
other could not obtain a driver’s license. Nevertheless, on his 18th birthday,
he borrowed his father’s car and took his girlfriend for a ride. With his 10%
vision in one eye, he was able to stay in the correct lane and avoid
oncoming traffic, but he failed to see a jogger on the edge of the highway.
The teenager’s car hit the jogger, causing serious bodily injury.
If the jogger prevails in a negligence suit against the teenager, it will be
because:
(A)The teenager failed to exercise ordinary and reasonable care under the
circumstances.
(B)The teenager failed to exercise the amount of care that an 18-year-old of
like education, intelligence, and experience would have exercised.
(C)The teenager failed to exercise the ordinary and reasonable care that a
person with the teenager’s disability would have exercised.
(D) The teenager violated the law when he drove without a license.

316
317
An inexperienced worker who was instructed to clean the floors of a store mixed
ammonia and chlorine bleach in a large pail. Both he and a customer who was
standing nearby were overcome by fumes and suffered lung damage. The
customer sued the worker, alleging negligence. In defense, the worker presented
uncontroverted evidence that he could not read the warning labels on the
containers and that, while he knew he was mixing ammonia and bleach, he had
never been made aware of the danger of mixing the two chemicals.
Nevertheless, the jury found him liable for the customer’s injuries.
If the worker challenges the verdict on appeal, the appellate court should:
(A)Uphold the verdict, because it was a determination that a reasonable person
should have known of the danger.
(B)Uphold the verdict, because it was a determination that the worker’s
evidence was not believed.
(C)Overrule the verdict, because it is inconsistent with the evidence.
(D)Overrule the verdict, because the worker’s lack of knowledge of the danger
should have been taken into account.

318
319
A driver traveling the speed limit in the evening on a quiet country road
rounded a curve and struck a bicyclist who was riding in the same lane. The
driver stopped the car and inspected the bicyclist, who had a broken leg. The
driver thought it best not to try to move the bicyclist, so he told him that he
would go to get help. The driver drove away and left the bicyclist by the side of
the road. After the driver had left the scene, he realized that he had forgotten
his wife’s birthday, so he stopped to buy a gift and hurried home. He did not
remember the bicyclist until a few hours later, but assumed that by that time
someone would have come along to render assistance. However, the bicyclist
was not rescued until the following morning. By then, he had contracted
pneumonia as a result of exposure.
The bicyclist sued the driver to recover damages for his broken leg and the
pneumonia.
If the jury finds that the driver was not negligent in his operation of his
automobile, the bicyclist will most likely:
(A)Recover for both the leg injury and the pneumonia.
(B)Recover for the leg injury but not the pneumonia.
(C)Recover for the pneumonia but not the leg injury.
(D) Not recover for either the leg injury or the pneumonia.
320
321
A landowner owned several dozen acres of mountain land near a national
forest. A plaintiff who was injured by a condition on the owner’s land
brought an action for personal injury against the landowner.
The plaintiff is most likely to win if she was:
(A)A 10-year-old trespasser who was swept onto some rocks while
attempting to cross a swiftly owing river.
(B)A five-year-old trespasser who fell into a mineshaft from which the
owner had removed all warning signs, but the plaintiff was not attracted
onto the owner’s land because of the mineshaft.
(C)A five-year-old trespasser who inadvertently stepped into a badger hole
that was obscured in the undergrowth.
(D)A 10-year-old niece visiting the landowner who stepped into a badger
hole that the landowner did not know was present but that could have
been discovered by inspection.
322
323
A tenant invited a friend over for dinner. On his arrival, the friend stepped
on a split board on the front steps and the board broke, causing him to
lose his balance and break his ankle.
If the friend sues the tenant for his injuries and does not prevail, it will be
because:
(A)In the lease, the landlord had undertaken the duty to discover and repair
dangerous conditions on the premises.
(B) The friend arrived an hour earlier than his invitation specified.
(C) The friend should have noticed the dangerous condition himself.
(D)The tenant had stayed beyond the lease term and she no longer had the
legal right to occupy the premises.

324
325
A camper at a state park built a camp fire within a fire ring on a calm day according to
approved procedures. Just as a sudden strong wind arrived and blew some embers onto
the grass, a large bear came out of the woods and charged at the camper. The camper
ran to his car, which was some distance away, with the bear in close pursuit. By the time
the bear left and the camper was able to exit the car and summon assistance, the
embers in the grass had started a brush fire. The fire destroyed another camper’s
equipment and automobile at a nearby campsite before it could be extinguished. The
other camper sued the camper who started the fire. At trial, the parties stipulated to the
above facts. The plaintiff introduced into evidence a state statute that prohibited leaving
any camp fires unattended and required them to be extinguished immediately if any
embers were blown out of the fire ring. At the conclusion of the proofs, both parties
moved for a directed verdict.
The court should:
(A)Grant the plaintiff’s motion, because the statute was intended to prevent the type of
harm that occurred, making the statutory standard applicable.
(B)Grant the plaintiff’s motion, because a brush fire caused by a camp fire does not
ordinarily happen in the absence of negligence by the camper.
(C)Grant the defendant’s motion, because the plaintiff has not established a prima facie
case of negligence.
(D)Deny both motions, because the jury should make the factual determination of
whether the defendant was negligent.
326
327
The plaintiff was driving inattentively when she had to swerve to avoid two
other negligently driven vehicles at a busy intersection, and her car struck a
light pole. The plaintiff, who was the only driver injured, sued one of the
other drivers to recover damages in a jurisdiction that has adopted pure
comparative negligence. The jury determined that she suffered injuries of
$100,000 and was 50% at fault.
If the plaintiff is awarded a recovery of only $25,000 from the defendant, it
will be because:
(A) The defendant’s fault was less serious than that of the other tortfeasor.
(B)The plaintiff’s fault was as great as the total negligence of the other two
drivers combined.
(C)The jurisdiction applies contribution based on a pro rata approach rather
than proportional fault.
(D) The jurisdiction has abolished joint and several liability.
328
329
A pedestrian crossed the street at a crosswalk without looking for oncoming
traffic. He was struck first by a car and then by a truck. The pedestrian
sued both the driver of the car and the driver of the truck for negligence.
The jury determined that the pedestrian was 60% at fault, the driver of the
car 30%, and the truck driver 10%. The jury also determined that the
pedestrian suffered damages of $100,000. The driver of the car is
insolvent.
In a pure comparative negligence jurisdiction retaining traditional joint
liability rules, how much can the pedestrian collect from the driver of the
truck?
(A)
Nothing.
(B)
$10,000.
(C) 330
$40,000.
331
A trucker owned and operated a small truck which he used commercially to
haul dynamite to construction sites. Unbeknownst to the trucker, there was
a hidden defect in the latch that held the rear panel of the truck. The
trucker was hauling a load of dynamite one morning and exceeding the
speed limit when his truck struck a bump in the road, the latch
malfunctioned, and the rear panel of the truck flew open. One box of
dynamite fell out of the truck and struck a pedestrian, breaking her foot.
If the pedestrian sues the trucker under strict liability for her injuries, the
pedestrian will:
(A)Win, because hauling dynamite is an abnormally dangerous activity.
(B)Win, because the trucker was speeding while driving with the
dynamite.
(C)Lose, because the defect in the latch was not discoverable
upon reasonable inspection.
(D) Lose, because the dynamite did not explode. 332
333
After accounts of a confidential congressional hearing on a national security
matter were published, the chief counsel at the hearing made a statement to a
major newspaper accusing a popular network news anchorman of leaking the
story and endangering national security. The network immediately fired the
anchorman. When facts came to light a few weeks later showing that the
allegation was not true, the anchorman was rehired and restored to his
position.
The anchorman sued the newspaper for defamation, claiming compensatory and
punitive damages, and made allegations legally sufficient to sustain those
damages if proved. No affirmative defenses were allowed.
The best defense of the newspaper would be that:
(A) It was not negligent in printing the chief counsel’s remarks.
(B) The anchorman was restored to his position within a few weeks.
(C)The publication was not made with knowledge that it was false or with
reckless disregard for the truth.
(D) The statement was protected by the Speech and Debate Clause.

334
335
A famous comedian was asked by the host of a popular late night television talk
show what brand of cigars he smoked. He responded, “I smoke only [the
manufacturer’s] cigars, because they’re the best.”
Two weeks later the manufacturer of those cigars began a national advertising
campaign featuring billboards, posters for use in retail stores, and full-page ads
in high circulation magazines. The advertising featured a picture of the
comedian with the manufacturer’s cigar in his hand, and the copy quoted his
statement from the show. The manufacturer had not received the comedian’s
permission to use either his picture or the statement that had been made
during the interview.
Will the comedian prevail in an action against the manufacturer for using his
picture and statement?
(A)Yes, because the comedian has been defamed.
(B)Yes, because the comedian’s likeness was appropriated for a commercial
purpose without his consent.
(C)No, because the advertising accuratelyreflects what the comedian
said publicly before millions of television viewers.
(D)No, because the comedian’s appearance on television created an implied
consent to reasonable use of anything he might say.
336
337
A horse breeder owned a small but exceptionally well-tended horse farm for
many years. The county in which the farm was located had no zoning or land-
use regulations, but that had never been a problem until a half-acre plot of land
next to the farm was recently purchased by a salvage company. The company let
the weeds grow high on the land and it became littered with smelly, unsightly
garbage and rusting metal. The breeder complained to the company on several
occasions but was ignored. In addition, business started to taper off at the
breeding farm due to the noise, smells, and general disarray of the junkyard.
If the breeder brings an action for nuisance against the company, how will the
court rule?
(A)For the breeder, because the breeder was a property owner in the area long
before the company bought the lot and opened the business.
(B)For the breeder, if he can show a substantial and unreasonable interference
with the use and enjoyment of his land.
(C)For the company, because it is using the land for legal purposes.
(D)For the company, unless the breeder can objectively demonstrate that the
value of the farm has declined.

338
339
Thank
you!

Good
luck!

340

You might also like